3/4 divided by 1/3 as a fraction

Answers

Answer 1

Answer:

[tex]\frac{9}{4}[/tex]

Step-by-step explanation:

Answer 2

Answer:

   [tex]\frac{9}{4}[/tex]

Step-by-step explanation:

[tex]\frac{3}{4}[/tex] ÷ [tex]\frac{1}{3}[/tex]

[tex]\frac{3}{4}[/tex] × [tex]\frac{3}{1}[/tex]  = [tex]\frac{9}{4}[/tex] ( to divide we make the other fraction reciprocal)

[tex]\frac{9}{4}[/tex] as mixed fraction= [tex]\frac{1}{4}{2}[/tex]


Related Questions

pls help Are the following lines parallel, perpendicular, or neither?

y = 2/3x − 4

y = −3/2x − 7

Responses


Parallel

Perpendicular

Neither

Answers

Answer:

Perpendicular.

Step-by-step explanation:

To determine whether the two lines are parallel, perpendicular, or neither, we need to compare their slopes.

The slope-intercept form of a line is y = mx + b, where m is the slope and b is the y-intercept. So we can rewrite the given equations in this form

y = 2/3x - 4 ==> slope = 2/3

y = -3/2x - 7 ==> slope = -3/2

Two lines are parallel if and only if their slopes are equal. Therefore, since the slopes of the two lines are different (2/3 and -3/2), they cannot be parallel.

Two lines are perpendicular if and only if their slopes are negative reciprocals of each other. That is, if the product of their slopes is -1. Therefore, we can check if the product of the slopes of the two lines is -1

(2/3) * (-3/2) = -1

Since the product of the slopes is -1, the two lines are perpendicular.

Therefore, the answer is: perpendicular.

A local winery wants to create better marketing campaigns for its white wines by understanding its customers better. One of the general beliefs has been that higher proportion of women prefer white wine as compared to men. The company has conducted a research study in its local winery on white wine preference. Of a sample of 400 men, 120 preferred white wine and of a sample of 500 women, 170 preferred white wine. Using a 0.05 level of significance, test this claim.INPUT Statistics required for computation170 = Count of events in sample 1500 = sample 1 size120 = Count of events in Sample 2400 = sample 2 size0.05 = level of significance0 = hypothesized differenceOUTPUT Output valuesSample 1 Proportion 34.00%Sample 2 Proportion 30.00%Proportion Difference 4.00%Z α/2 (One-Tail) 1.645Z α/2 (Two-Tail) 1.960Standard Error 0.031Hypothesized Difference 0.000One-Tail (H0: p1 − p2 ≥ 0)Test Statistics (Z-Test) 1.282p-Value 0.900One-Tail (H0: p1 − p2 ≤ 0)Test Statistics (Z-Test) 1.282p-Value 0.100Two-Tail (H0: p1 − p2 = 0)Test Statistics (Z-Test) 1.276p-Value 0.202Group of answer choicesThis is a one-tail test and the data does support the claim that higher proportion of women prefer white wine as compared to men.This is a one-tail test and the data does not support the claim that higher proportion of women prefer white wine as compared to men.This is a two-tail test and the data does support the claim that higher proportion of women prefer white wine as compared to men.This is a two-tail test and the data does not support the claim that higher proportion of women prefer white wine as compared to men.Question 2. Based on the study results presented in the last question, what is the upper bound for the proportion differences between women and men for a 95% confidence interval?(Note: Please enter a value with 4 digits after the decimal point. For example, if you computed an upper boundary of 23.456% or .23456, you would enter it here in decimal notation and round it to four digits, thus entering .2346).

Answers

Answer:

235.65

Step-by-step explanation:

Un robot salta sobre una recta numérica iniciando en el cero. Las reglas del juego son

las siguientes si mira hacia tu mano derecha, la longitud de cada salto se expresa

con un número positivo y, si mira hacia tu mano izquierda, con un número negativo.

También: si salta de frente (o sea, en la dirección que está mirando), el número

de saltos se expresa con un número positivo, y si salta de espaldas (es decir, en

la dirección contraria a la que está mirando), el número de saltos se considera con

un número negativo.

Observa los siguientes saltos del robot, su posición final en cada caso y contesta en tu

cuaderno las preguntas

Answers

Each subsequent jump will be exactly one unit longer in length than the one before it after the initial jump, which can be one unit in length. Each jump has the option of going either left or right.

The translation of the question is

A robot jumps on a number line starting at zero. The rules of the game are the following if you look at your right hand, the length of each jump is expressed with a positive number and, if facing your left hand, with a negative number.

Also: if you jump straight ahead (that is, in the direction you are facing), the number number of jumps is expressed as a positive number, and if you jump backwards (i.e., on the opposite direction to the one you are looking at), the number of hops is considered with a negative number. Observe the following jumps of the robot, its final position in each case and answer in your notebook the questions

When closely examined, it is simple to conclude that:

If you have consistently jumped in the appropriate direction, you will arrive at point p = 1 + 2 + 3 + 4 +... + n after n jumps.

You would be at point p - 2k in any of these n leaps if you leapt left in the kth hop (k=n) rather than right.

Moreover, after n leaps, you can be anywhere between n * (n + 1) / 2 and -(n * (n + 1) / 2) with the same parity as n * (n + 1) / 2 by carefully selecting which jumps to go left and which to go right.

The trick is to imitate jumping while keeping the aforementioned facts in mind. Always jump to the right, and if at some time you arrive at a location that has the same parity as X and is at or beyond X, you'll know the answer.

To know more about positive number , click the below link

https://brainly.com/question/13165973

#SPJ4

Terri pays a monthly cell phone fee of $10. She pays 5 cents for each minute that she talks. If Terri does not make any calls, what would her bill be?

Answers

Answer:

$10

Step-by-step explanation:

We Know

Terri pays a monthly cell phone fee of $10. She pays 5 cents for each minute that she talks.

Let C be the total cost, and x be the number of minutes she talks; we have the equation.

C = 0.05x + 10

If Terri does not make any calls, what would her bill be?

C = 0.05(0) + 10

C = $10

So, her bill will be $10

g in acid base reactions, the hydrogen ions from the acid and the hydroxide ions from the base neutralize each other. khp has one ionizable hydrogen; this means that one mole of sodium hydroxide neutralizes one mole of khp. from experiment 1, calculate the exact molarity of the sodium hydroxide. (hint: use the mass of khp and do a stoichiometry problem.....) tip: khp is not the chemical formula. khp stands

Answers

In the following question, among the conditions given, the statement is said to be, the exact molarity of the NaOH solution is 0.0960 M.

The question is asking to calculate the exact molarity of the sodium hydroxide from Experiment 1.
KHP stands for potassium hydrogen phthalate, and one mole of sodium hydroxide (NaOH) will neutralize one mole of KHP. To solve the problem, use the mass of KHP and a stoichiometry problem.
First, calculate the number of moles of KHP:
Moles KHP = (Mass KHP (g) / Molar Mass KHP (g/mol))
Then, calculate the moles of NaOH:
Moles NaOH = (Moles KHP * Mole Ratio NaOH/KHP)
Finally, calculate the molarity of NaOH:
Molarity NaOH = (Moles NaOH / Volume NaOH (L))

For more such questions on NaOH

https://brainly.com/question/15973235

#SPJ11

There is a 0.99962 probability that a randomly selected 28​-year-old female lives through the year. An insurance company wants to offer her a​ one-year policy with a death benefit of ​$500,000. How much should the company charge for this policy if it wants an expected return of ​$400 from all similar​ policies?

Answers

In order to expect a return on $400 from across all policies of a similar nature, the insurance firm should charge the policy for about $501.88.

How then do we return a value?

Return[expr] leaves control structures that are present during a function's definition and returns the value expression for the entire function. Even if it comes inside other functions, yield takes effect as quickly as it is evaluated. Functions like Scan can use Return inside of them.

Since p is the chance that the 28-year-old woman survives the year and is given as 0.99962, we can enter this number into the equation for n as follows: n = 400(0.99962)/500,400 n 0.799

In light of this, the insurance provider should impose a premium of: Premium = 400/n

$501.88 is the premium ($Premium = 400/0.799)

To know more about return visit:

https://brainly.com/question/28562900

#SPJ1

Help due soon !!!!!!!!!

Answers

An expression for the length of the rectangle in terms of A is [tex]$\boxed{L=x+5}$[/tex]

How to find the expression?

We are given that the area of a rectangle is [tex]$A=x^2+x-15$[/tex], and we want to find an expression for the length of the rectangle in terms of A.

Recall that the area of a rectangle is given by the formula: [tex]$A=L\cdot W$[/tex], where L is the length and W is the width. We can use this formula to write L in terms of A and W as [tex]$L=\frac{A}{W}$[/tex].

We know that the rectangle has a length and a width, so we need to find an expression for the width W in terms of A. We can rearrange the given formula for A to solve for W:

[tex]&& \text{(substitute }L=x+5\text{)}[/tex]

[tex]W&=\frac{x^2+x-15}{x+5} && \text{(divide both sides by }x+5\text{)}[/tex]

Now that we have an expression for W in terms of A, we can substitute it into our expression for L to get:

[tex]L&=\frac{A}{W}[/tex]

[tex]&=\frac{x^2+x-15}{\frac{x^2+x-15}{x+5}} && \text{(substitute the expression we found for }W\text{)}\&=x+5[/tex]

Therefore, an expression for the length of the rectangle in terms of A is [tex]$\boxed{L=x+5}$[/tex]

To know more about rectangle visit:

https://brainly.com/question/29123947

#SPJ1

Jerry, Jack and Sophie are all hoping to save​ money! Jerry thinks saving money in a shoe box in his closet every month is a good idea. He decides to start with​ $125, and then save​ $50 each month. Jack was given​ $3520 from his​ Grandma, and decides to put the money
into an account that has a​ 6.5% interest rate that is compounded annually. Sophie has earned​ $3500 working at the movie theater decides to put her money in the bank in an account that has a​ 7.05% interest rate that is compounded continuously
Part 1​: Describe the type of equation that models​ Jerry’s situation. Create that equation of​ Jerry’s situation. Using the equation you​ created, how much money will be in​ Jerry’s account after 3​ years? 10​ years?
Think​: What do I know and what does it​ mean? What plan am I going to​ try?

PLEASE HELP!!!!!

Answers

Jerry will have $1825 in his account after 3 years and Jerry will have $6125 in his account after 10 year when compounded.

What is simple interest?

Simple interest is computed just using the principle, which is the initial sum borrowed or put into an investment. The interest rate is constant throughout time and solely applies to the principal sum. Short-term loans or investments frequently employ simple interest.

The given situation can be modeled as a linear equation given by:

y = mx + c

For Jerry we have:

y = 50x + 125

For 3 years = 36 months we can substitute x = 36:

y = 50(36) + 125

y = 1825

For x = 10:

y = 50(120) + 125

y = 6125

Hence, Jerry will have $1825 in his account after 3 years and Jerry will have $6125 in his account after 10 year.

Learn more about simple interest here:

https://brainly.com/question/30824126

#SPJ1

find the percent of the discount: a $30 board game on sale for 21​

Answers

well, we know the discount is just 30 - 21 = 9, so hmm if we take 30(origin amount) to be the 100%, what's 9 off of it in percentage?

[tex]\begin{array}{ccll} Amount&\%\\ \cline{1-2} 30 & 100\\ 9& x \end{array} \implies \cfrac{30}{9}~~=~~\cfrac{100}{x} \\\\\\ 30x=900\implies x=\cfrac{900}{30}\implies x=30[/tex]

one hundred students were asked whether they liked certain candy flavors. it was found that liked cherry, liked coconut, and liked both flavors. what's the probability that a randomly selected student...

Answers

The probability that a randomly selected student likes coconut is 0.6.

Step-by-step explanation:

Given that,

The number of students = 100 The number of students who liked cherry = 30 The number of students who liked coconut = 60 The number of students who liked both flavors = 10

Now we have to find the probability that a randomly selected student likes coconut.

P (student likes coconut) = the number of students who liked coconut / total number of students

= 60/100

= 0.6

Therefore, the probability that a randomly selected student likes coconut is 0.6.

See more about probability at: https://brainly.com/question/13604758

#SPJ11

Suppose that a phone originally sold for $800 loses 3/5 of its value each year after it is released. After 2 years, how much is the phone worth?A. $800B. $1333C. $128D. $288​

Answers

The phone is worth $128 after 2 years, which is option C.

What is exponential decay ?

Exponential decay is a decrease in a quantity over time where the rate of decay is proportional to the current value. In this case, the value of the phone decreases by 3/5 each year after it is released. This means that the value after one year is 2/5 of the original value, and the value after two years is (2/5) times (2/5) of the original value. Exponential decay is a common phenomenon in many areas of science and mathematics, including radioactive decay, population growth and decay, and financial investments.

Calculating the worth of the phone :

The phone is not worth the same amount after 2 years, as it loses 3/5 of its value each year. We need to calculate its worth after 2 years.

Let's use the formula for exponential decay: [tex]A = A_0(1 - r)^t[/tex], where A is the final amount, [tex]A_0[/tex] is the initial amount, r is the decay rate, and t is the time elapsed.

In this case, the initial amount is $800, the decay rate is 3/5, and the time elapsed is 2 years. Substituting these values into the formula, we get:

[tex]A = 800(1 - 3/5)^2[/tex]

[tex]A = 800(2/5)^2[/tex]

[tex]A = 800(4/25)[/tex]

[tex]A = 128[/tex]

Therefore, the phone is worth $128 after 2 years, which is option C.

To know more about exponential decay visit :

brainly.com/question/2193799

#SPJ1

I don’t know what I’m doing

Answers

Answer:

29.12 square cm

Step-by-step explanation:

Area of equilateral triangle:

     Side = a = 8.2 cm

    [tex]\boxed{\bf Area \ of \ equilateral \ triangle = \dfrac{\sqrt{3}}{4}a^2}[/tex]

                                                       [tex]\sf = \dfrac{\sqrt{3}}{4}*8.2*8.2\\\\= \sqrt{3}*4.1 * 4.1\\\\= 1.732 * 4.1 *4.1\\\\= 29.12 \ cm^2[/tex]

Mrs banks wants to make 44 quarts of jelly with 70 pounds of fruit if each gallon of jelly costs 6. 5 pounds of fruit will she of enough fruit and will there be extra

Answers

Mrs. Banks has enough fruit to make the 44 qt of jelly she wants, but she will have 4 lb of leftover fruit.

Here we have to use the arithmetic operations. First, we need to convert the total quantity of jelly to gallons since we have the amount of fruit needed per gallon. One gallon is equal to 4 quarts, so 44 quarts is equal to 11 gallons.

Next, we can calculate how much fruit is needed for 11 gallons of jelly by multiplying the amount of fruit needed per gallon by the number of gallons

11 gallons x 6 lb of fruit per gallon = 66 lb of fruit needed

Since Mrs. Banks only has 70 lb of fruit, she has enough to make the 44 qt of jelly she wants, but she will have 4 lb of leftover fruit:

70 lb of fruit - 66 lb of fruit needed = 4 lb of leftover fruit

Learn more about Arithmetic operations here

brainly.com/question/30553381

#SPJ4

The given question is incomplete, the complete question is:

Mrs. Banks wants to make 44 qt of jelly with 70 lb of fruit. If each gallon of jelly needs 6 lb of fruit, will

she have enough fruit? How much leftover fruit does she have, or how much extra fruit is needed?


Jackson owns a heating and cooling company. He is going to install a new furnace into a customer's house, but he must determine the volume of airflow
needed in order to select the best size of furnace. Find the volume of the house sketched below, where H1-10 ft., H2=9 feet, L=30 feet, and W=20 feet.
Volume of a rectangular prism is V=WLH (width x length x height) Volume of a triangular Prism is:
V =
a.ch
A
A target has a bull's-eye with a d X
O mathwarehouse.com
h (in this case a-9, c= 20, h= 30)
15

Answers

[tex]8700[/tex] cubic feet of airflow are required for the entire home. Jackson may utilize this information to determine the best furnace size for the customer's requirements.

Volume explain: What is it?

Volume is the quantity of space an object occupies, whereas capacity is a measurement of the substance—such as a solid, liquid, or gas—that an object can hold. While capacity may be measured in virtually any other unit, such as liters, gallons, pounds, etc., volume was determined in cubic units.

What are volume and what is its unit?

Volume, which is measured in cubic units, is the three dimensional space inhabited by material or surrounded by a surface. The cubic centimeter (m3), a derived unit, is the SI unit for volume.

Volume of the first section with height [tex]H1=10[/tex] ft:

[tex]V1 = WLH1 = (20 ft)(30 ft)(10 ft) = 6000[/tex] cubic feet

Volume of the second section with height [tex]H2=9[/tex] ft:

[tex]V2 = (1/2)WLH2 = (1/2)(20 ft)(30 ft)(9 ft) = 2700[/tex]cubic feet

Total volume of house,

[tex]V total = V1 + V2 = 6000[/tex] cubic feet + [tex]2700[/tex] cubic feet [tex]= 8700[/tex] cubic feet

Therefore, the volume of airflow needed for the house is [tex]8700[/tex] cubic feet.

To know more about volume visit:

https://brainly.com/question/1578538

#SPJ1

commuting times for employees of a local company have a mean of 63.6 minutes and astandard deviation of 2.5 minutes. what does chebyshev's theorem say about thepercentage of employees with commuting times between 58.6 minutes and 68.6 minutes?

Answers

According to Chebyshev's theorem, at least 75% of the employees will have commuting times that fall within 2 standard deviations of the mean, or between 58.6 minutes and 68.6 minutes.

Chebyshev's theorem states that for any set of data, regardless of its distribution, a certain percentage of the data lies within a certain number of standard deviations from the mean. Specifically, Chebyshev's theorem states that for any data set, at least 1 – 1/k² of the data values will lie within k standard deviations of the mean, where k is any number greater than 1. If k=2, at least 75% of the data values lie within 2 standard deviations of the mean. If k=3, at least 89% of the data values lie within 3 standard deviations of the mean.

Therefore, for a data set with a mean of 63.6 minutes and a standard deviation of 2.5 minutes, we can use Chebyshev's theorem to determine that at least 75% of the employees will have commuting times that fall within 2 standard deviations of the mean, or between 58.6 minutes and 68.6 minutes.

Learn more abut Chebyshev's theorem here: https://brainly.com/question/5179184

#SPJ11

Analyze the proportion below and complete the instructions that follow. Use a model to find the missing value in the proportion. A. 4 B. 5 C. 10 D. 22 Please select the best answer from the choices provided A B C D

Answers

Step-by-step explanation:

The area of a rectangle is 1,872 ft2. The ratio of the length to the width is 9:13. Find the perimeter of the rectangle.

176 ft

You want to make a scale drawing of your bedroom to help arrange your furniture. You decide on a scale of 3 in. = 2 ft. Your bedroom is a 12 ft by 14 ft rectangle. What should the dimensions of your drawing be?

18 in. by 21 in.

If 5/y + 7/x=24 and 12/y + 2/x=24, find the ratio of x to y.

5/7

Simplify the ratio 8ft/12in. Use the conversion 12 in. = 1 ft.

8/1

Analyze the proportion below and complete the instructions that follow.

2x+5/3 = x-5/4

-7

If a+b/2a-b = 5/4 and b/a+9 = 5/9, find the value of b.

30

Analyze the ratio below and complete the instructions that follow.

$30:$6

Simplify the ratio.

5:1

If 14/3 = x/y then 14/x =

3/y

Analyze the diagram below and complete the instructions that follow.

In the diagram, AB:BC is 3:4 and AC = 42. Find BC.

24

Analyze the diagram below and complete the instructions that follow.

If AB:BC is 3:11, solve for x.

9

If a, b, c, and d are four different numbers and the proportion a/b = c/d is true, which of the following is false?

b/a = c/d

Analyze the diagram below and complete the instructions that follow.

Find the ratio of the width to the length of the rectangle, then simplify the ratio. Use the conversion 100 cm = 1 m.

3/4

Simplify the ratio 3 gal./24 qt. Use the conversion 4 qt = 1 gal.

1/2

The area of a rectangle is 4,320 ft2. The ratio of the length to the width is 6:5. Find the length of the rectangle.

72 ft

Analyze the diagram below and complete the instructions that follow.

Given that CB/CA = DE/DF, find BA.

10.5

Analyze the proportion below and complete the instructions that follow.

2/3 = 8/x

3, 8

Analyze the diagram below and complete the instructions that follow.

Are the polygons shown here similar? Justify your answer. The images are not drawn to scale.

Yes, PQR ~TSV with a scale factor of 1:√3

All __________ are similar.

squares

Analyze the diagram below and complete the instructions that follow.

Determine which 2 triangles are similar to each other. The images are not drawn to scale.

GHI ~ JKL

Analyze the diagram below and complete the instructions that follow.

Pentagon PQRST ~ pentagon XYZVW. Find the value of b. The images are not drawn to scale.

3

Analyze the diagram below and complete the instructions that follow.

If ABC ~ XYZ, find XY. The images are not drawn to scale.

24

ABC is a right triangle. The legs of ABC are 9 ft and 12 ft. The shortest side of XYZ is 13.5 ft, and ABC ~ XYZ How long is the hypotenuse of XYZ?

22.5 ft

The store sells a television for $1000. customers can choose to receive 10% discount and pay it off at a simple interest rate of 4% or they can choose to pay the full price and pay it off in 3 years with no interest. which option is better

Answers

Option 1 with the discount and 4% simple interest has a total cost of $972, while Option 2 with no discount and no interest has a total cost of $1000. Option 1 is the better choice as it has a lower total cost.

What is simple interest?

Simple interest is a type of interest that is calculated on the original principal amount of a loan or investment. It is a fixed percentage of the principal amount that is paid by the borrower or earned by the lender over a specific period of time.

According to question:

To compare the two options, we need to calculate the total cost of each option and compare them.

Option 1: 10% discount and pay off with 4% simple interest

The discount reduces the price of the television to $1000 x 0.9 = $900. If the customer chooses to pay it off at 4% simple interest, the total cost would be:

Total cost = $900 + ($900 x 0.04 x 3) = $972

Option 2: Full price and pay off in 3 years with no interest

The total cost of this option would be simply the full price of $1000 paid over 3 years, so:

Total cost = $1000 / 3 = $333.33 per year x 3 years = $1000

Comparing the two options, we see that Option 1 with the discount and 4% simple interest has a total cost of $972, while Option 2 with no discount and no interest has a total cost of $1000. Therefore, Option 1 is the better choice as it has a lower total cost.

To know more about simple interest visit:

https://brainly.com/question/29210957

#SPJ1

The complete question is: The store sells a television for $1000. customers can choose to receive 10% discount and pay it off at a simple interest rate of 4% or they can choose to pay the full price and pay it off in 3 years with no interest. which option is better?

Option 1 with the discount and 4% simple interest.

Option 2 with no discount and no interest.

A cruise ship compartment can hold 440 pieces of luggage. If a ship had 42 compartments,how many pieces of luggage can it hold?

Answers

Answer: 18480

i used a calculator :)

I need help with this

Answers

sum area = -3x - 6y + 12 and product area = -36x - 72y.

what is rectangle?

A rectangle is a geometric shape that is defined as a four-sided flat shape with four right angles (90-degree angles) and opposite sides that are parallel and equal in length.

The area of a rectangle is given by the product of its length and width. Assuming that the length of the rectangle is given by -3x - 6y and its width is 12, we can express the area in terms of a sum and a product as follows:

Sum:

Area = length x width

Area = (-3x - 6y) + 12

Area = -3x - 6y + 12

Product:

Area = length x width

Area = (-3x - 6y) x 12

Area = -36x - 72y

Note that the product expression is not equal to the sum expression. This is because we used different assumptions for the length of the rectangle in each case.

Therefore, sum area = -3x - 6y + 12 and product area = -36x - 72y.

To learn more about rectange from the. given link:

https://brainly.com/question/29123947

#SPJ1

Sophie invested $92,000 in an account paying an interest rate of 6 1/8% compounded

continuously. Damian invested $92,000 in an account paying an interest rate of 6 5/8%

compounded monthly. After 14 years, how much more money would Damian have in

his account than Sophie, to the nearest dollar?

Answers

Answer:

Step-by-step explanation:

To solve this problem, we need to use the formula for compound interest:

A = P*e^(rt)

where A is the final amount, P is the principal (initial investment), e is the base of the natural logarithm (approximately 2.71828), r is the interest rate (expressed as a decimal), and t is the time (in years).

For Sophie's account, we have:

P = $92,000

r = 6 1/8% = 0.06125 (as a decimal)

t = 14 years

A = 92000*e^(0.06125*14)

A = $219,499.70 (rounded to the nearest cent)

For Damian's account, we have:

P = $92,000

r = 6 5/8% = 0.06625/12 = 0.005521 (as a monthly decimal rate)

t = 14*12 = 168 months

A = 92000*(1+0.005521)^168

A = $288,947.46 (rounded to the nearest cent)

Now we can subtract Sophie's final amount from Damian's final amount to find the difference:

Difference = $288,947.46 - $219,499.70

Difference = $69,447.76

Therefore, Damian would have about $69,448 more in his account than Sophie, to the nearest dollar.

i need help!
f(x)=3x3+9x2-12x
g(h)=x-1
h(x)=3x2+12x

Answers

Expression which Defines function h is as follows :

[tex]f(g(h(x))) = 81x^9 + 2916x^8 - 351x^7 - 81762x^6 - 13230x^5 + 705228x^4 - 127752x^3 - 348744x^2 + 12480x + 20736.[/tex]

What does a function mean to you?

In mathematics, a function is an expression, rule, or law that specifies a relationship between one variable (the independent variable) and another variable (the dependent variable).

To find f(g(h(x))), we must first find h(x), then plug it into g(h(x)), and finally into f. (x).

[tex]h(x) = 3x^2 + 12x[/tex]

[tex]g(h(x)) = h(x) - 1 = (3x^2 + 12x) - 1 = 3x^2 + 12x - 1[/tex]

[tex]f(g(h(x))) = 3(3x^2 + 12x - 1)^3 + 9(3x^2 + 12x - 1)^2 - 12(3x^2 + 12x - 1)[/tex]

Simplifying this expression is time-consuming, but we can use the binomial theorem to expand each term and then combine like terms to get:

[tex]f(g(h(x))) = 81x^9 + 2916x^8 - 351x^7 - 81762x^6 - 13230x^5 + 705228x^4 - 127752x^3 - 348744x^2 + 12480x + 20736[/tex]

Therefore, [tex]f(g(h(x))) = 81x^9 + 2916x^8 - 351x^7 - 81762x^6 - 13230x^5 + 705228x^4 - 127752x^3 - 348744x^2 + 12480x + 20736.[/tex]

To know more about Function visit:

brainly.com/question/14996337

#SPJ1

Can anyone solve this problem please? Thanks!

Answers

The trapezoid has a surface area of 480 square units.

What is the measurement for a trapezoid's area?

So, a trapezoid measured in feet offers an area in square feet; one measured in millimetres gives an area in square centimetres; and so on. If it's simpler for you, you can add the lengths of the bases and then divide the total by two. Keep in mind that multiplication by 12 is equivalent to dividing by 2.

We must apply the formula for a trapezoid's area to this issue in order to find a solution:

[tex]A = (1/2) * (a + b) * h[/tex]

where h is the trapezoid's height (or altitude) and a and b are the lengths of its parallel sides.

The values for a, b, and h are provided to us, allowing us to change them in the formula:

A = (1/2) * (20 + 60) * 12

A = (1/2) * 80 * 12

A = 480 square units

To know more about trapezoid visit:-
https://brainly.com/question/8643562

#SPJ1

what are the roots of 2x^2+10x+9=2x

Answers

The roots of the equation 2x² + 10x + 9 = 2x does not exist i.e no real roots

Calculating the roots of the equation

To find the roots of the given quadratic equation 2x² + 10x + 9 = 2x, we can start by rearranging the equation to the standard form of a quadratic equation

2x² + 10x + 9 - 2x = 0

Simplifying the left-hand side, we get:

2x² + 8x + 9 = 0

Now, we can use the quadratic formula to find the roots of the equation:

x = (-b ± √(b² - 4ac)) / 2a

where a = 2, b = 8, and c = 9.

Substituting these values into the formula, we get:

x = (-8 ± √(8² - 4(2)(9))) / 2(2)

Simplifying the expression under the square root, we get:

x = (-8 ± √-8) / 4

The square root of -8 is not a real number

So, the equation has no real root

Read more about quadratic equation at

https://brainly.com/question/24334139

#SPJ1

Subtract the given equation

3x-(4x-11)

Answers

Answer:

3x - (4x - 11) = 3x - 4x + 11 = -x + 11

Step-by-step explanation:

Question 9(Multiple Choice Worth 2 points)
(Irrational Numbers LC)
Describe in words where √63 would be plotted on a number line.
O Between 3 and 4, but closer to 3
O Between 3 and 4, but closer to 4
O Between 2 and 3, but closer to 2
O Between 2 and 3, but closer to 3

Answers

It would be plotted Between 3 and 4, but closer to 4

Answer:

Between 3 and 4, but closer to 4

Use the following circle to find the indicated measure.

MK
is a diameter.

Find m ∠
LKM

Answers

The answer of the given question based on finding the m∠LKM from the given circle the answer is the measure of ∠LKM is 140° degrees.

What is Diameter?

In geometry,  diameter of circle is  line segment that passes through  center of  circle and has both endpoints on circle. The diameter is the longest chord (line segment connecting two points on  circumference) of  circle. The length of  diameter is twice the length of radius, which is distance from the center of  circle to any point on  circumference.

The diameter i important property of a circle and is used to calculate other properties, like the circumference and area of the circle

Since MK is a diameter of the circle, it passes through the center of the circle, which we can label as point O. Therefore, ∠LKM is an inscribed angle that intercepts arc LM.

By the Inscribed Angle Theorem, we know that the measure of an inscribed angle is equal to half the measure of the arc that it intercepts. Therefore, to find the measure of ∠LKM, we need to find the measure of arc LM.

We are given that the measure of arc LK is 100° degrees. Since arc LM is the sum of arcs LK and KM, and MK is a diameter (so arc KM is also a semicircle with a measure of 180 degrees), we can write:

m(arc LM) = m(arc LK) + m(arc KM) = 100 + 180 = 280° degrees

Therefore, the measure of ∠LKM is:

m∠LKM = 1/2 * m(arc LM) = 1/2 * 280 = 140° degrees

So the measure of ∠LKM is 140° degrees.

To know more about  Inscribed Angle Theorem visit:

https://brainly.com/question/5436956

#SPJ1

10 is 30% of what number?
Solution:
Part:
whole
percent:

Answers

The number 10 is 30% of the following number: 33.33.

What is a percentage?

In Mathematics, a percentage can be defined as any number that is expressed as a fraction of hundred (100). This ultimately implies that, a percentage indicates the hundredth parts of any given number.

How to calculate the percentage of a number?

In order to determine the whole number which 10 is 30%, we would apply the following mathematical expression (formula);

Quantity = percent × number

Substituting the given parameters into the percentage and quantity formula, we have the following;

10 = 30/100 × number

10 = 0.3  × number

Number = 10/0.3

Number = 33.33.

Read more on percentage here: brainly.com/question/28009735

#SPJ1

Which expression is equivalent to 6(w+7)?

Answers

The expression that is equivalent to 6(w + 7) is 6w + 42.

What is an expression?

An expression is a combination of numbers, variables, and mathematical operations (such as addition, subtraction, multiplication, and division) that are grouped together to represent a mathematical quantity or relationship.

What is distributive property?

The distributive property is a property of multiplication that allows us to multiply a single term by a sum or difference of terms. It states that:

a(b + c) = ab + ac and a(b - c) = ab - ac.

In other words, we can distribute the factor a to each term within the parentheses by multiplying it by each term, and then add or subtract the resulting products.

In the given question,

To simplify the expression 6(w + 7), we can use the distributive property of multiplication over addition, which states that:

a(b + c) = ab + ac

Using this property, we can expand the expression 6(w + 7) as:

6w + 6(7)

Simplifying the second term by multiplying, we get:

6w + 42

Therefore, the expression that is equivalent to 6(w + 7) is 6w + 42.

To know more about distributive property, visit:

https://brainly.com/question/6276874

#SPJ1

For the value of the sum, enter an expression that gives the exact value, rather than entering an approximation.
A.-12 + 4 – 4/3 +4/9 – 4/27 +4/81 - … = -12/(1+1/3)
B.∑ (1/3)^n = 6*1/3^6(1/3^11-1)/(1/3-1)

Answers

a. The exact value of the sum of -12 + 4 – 4/3 +4/9 – 4/27 +4/81 - … = -12/(1+1/3)  is 12/7.

b.The exact value of the sum of∑ (1/3)ⁿ = 6*1/3⁶(1/3¹¹-1)/(1/3-1)  is 3/2.

A.-12 + 4 – 4/3 +4/9 – 4/27 +4/81 - …

This is an infinite geometric series with first term a = -12 and common ratio r = 4/(-3). The sum of an infinite geometric series is given by:

S = a / (1 - r)

Substituting the values of a and r, we get:

S = (-12) / [1 - (4/(-3))]

Simplify the denominator by multiplying both numerator and denominator by (-3):

S = (-12) / [-3 - 4]

S = (-12) / (-7)

S = 12/7

Therefore, the exact value of the sum is 12/7.

B. 6*1/3⁶(1/3¹¹-1)/(1/3-1)

This is a geometric series with first term a = 1 and common ratio r = 1/3. The sum of a geometric series with n terms is given by:

S = a (1 - rⁿ) / (1 - r)

As n approaches infinity, rⁿ approaches zero and the sum converges to:

S = a / (1 - r)

Substituting the values of a and r, we get:

S = 1 / (1 - 1/3)

S = 3/2

Therefore, an expression that gives the exact value of the sum is 3/2.

Learn  more about value at https://brainly.com/question/1446179

#SPJ11

PLEASE HELP ME QUICKLY!

Answers

Step-by-step explanation:

it would mean that she made 53 batches of soap and 4 batches of lotion.

now, is it a solution ?

then both inequalities must be true with these values.

5×53 + 15×4 <= 325

265 + 60 <= 325

325 <= 325 correct

20×53 + 35×4 <= 1200

remember, 1 hour = 60 minutes.

1060 + 140 <= 1200

1200 <= 1200 correct

so, (53, 4) is the intersection point of both limit lines. and it is as such an extreme point and optimum.

Other Questions
Read the sentence.Michael Jordan, the basketball player, won six championships for the Chicago Bulls.The commas are used correctly in the sentence because the underlined element isA - a nonrestrictive phrase.B - semicolensC - compound modifiers. There muscles extend from the pubis and ischium anteriorly to the sacrum and coccyx posteriorly. They control the evacuation of waste materials from the digestive and urinary systems.A- muscles of the abdominal wallB- Muscles of the vertebral columnC- Muscles of the pelvic floorD- Muscles of the pharynxE- Muscles of the perineal floor Use these two constants for the question that follows:e = 1.6 10^19 Ck = 8.99 10^9 N m^2/C^2A positive charge and a negative charge are 10^15 m away from each other. Using Coulomb's law, which of the following is the electrical force between these two particles? 230 N 230 N 120 N 120 N What percentage decrease in profits from 2021 to 2022 would imply that profits were equal in 2020 and 2022? The food service manager conducted a random survey of 200 students to determine their preference for new lunch menu items. There are 1,500 students in the school. Select all the managers predictions that are supported by the data in pea plants, the allele for purple flowers, p, is dominant over the allele for white flowers, p. which conclusion about phenotype and genotype is possible?(1 point) responses A. if a plant has white flowers, then its genotype must be pp. B. if a plant has white flowers, then its genotype must be , pp, . C. if a plant has white flowers, then its genotype must be pp. D. if a plant has white flowers, then its genotype must be , pp, . E. if a plant has purple flowers, then its genotype must be pp. F. if a plant has purple flowers, then its genotype must be , pp, . G. if a plant has purple flowers, then its genotype must be pp. How did collections of living plants from around the world probably MOST help botanists to increase their knowledge about plants? A. by enabling them to experiment with diverse plants B. by allowing them to study plant anatomy more closely C. by making it possible to grow plants anywhere D. by building public support for botanical research 3. if you buy a share of apple stock today, assume the following is true: a) you will be able to sell it in 3 years for $75 and b) you will receive a $5 dividend at the end of each of those three years. a. if the prevailing interest rate is 10%, what is the value of a share of apple today? b. if the present price is $60, should you buy the share? Hideo and Hannah decide to go on a vacation. As a result, they withdraw $5,000 from their savings account to purchase $5,000 worth of traveler's checks. As a result of these changes,a.M1 increases by $5,000 and M2 decreases by $5,000.b.M1 decreases by $5,000 and M2 increases by $5,000.c.M1 increases by $5,000 and M2 stays the same.d.M1 and M2 stay the same. under which circumstances would you expect a person to demonstrate the highest level of conformity in solomon asch's research protocol? 50 POINTS PLS HELP AND BRAINLIEST The ideas of the Jacksonian Democrats are best represented by the phrase,A. *politics for the "common man."B. "opportunity for immigrants."C. "politics for the under represented."D. "voting rights for women * I need help please show your work Ideal Gas LabData:Complete the table to organize the data collected in this lab. Dont forget to record measurements with the correct number of significant figures.(Table attached below)Data Analysis:Create a separate graph of temperature vs. volume for each of the gas samples. You are encouraged to use graphing software or online tools to create the graphs; be sure to take screenshots of the graphs that also include your data.Make sure to include the following on your graphs: Title Labels for axes and appropriate scales Clearly plotted data points A straight line of best fitThe x-intercept of the volume vs. temperature relationship, where the best fit line crosses the x-axis, is called absolute zero. Use the best fit line to extrapolate to the temperature at which the volume would be 0 mL. Record this value. It is your experimental value of absolute zero.Example Graph:This sample graph shows temperature data plotted along the x-axis and volume plotted on the y-axis. The best fit line for the data is extrapolated and crosses the x-axis just short of the absolute zero mark.Calculations:1. The actual value for absolute zero in degrees Celsius is 273.15. Use the formula below to determine your percent error for both gas samples.|experimental value actual value| x 100 actual value2. If the atmospheric pressure in the laboratory is 1.2 atm, how many moles of gas were in each syringe? (Hint: Choose one volume and temperature pair from your data table to use in your ideal gas law calculation.)Conclusion:Write a conclusion statement that addresses the following questions:How did your experimental absolute zero value compare to the accepted value?Does your data support or fail to support your hypothesis (include examples)? Discuss any possible sources of error that could have impacted the results of this lab.How do you think the investigation can be explored further?Post-Lab Reflection QuestionsAnswer the reflection questions using what you have learned from the lesson and your experimental data. It will be helpful to refer to your chemistry journal notes. Answer questions in complete sentences.1. Why was the line of best fit method used to determine the experimental value of absolute zero?2. Which gas law is this experiment investigating? How does your graph represent the gas law under investigation? 3. Using your knowledge of the kinetic molecular theory of gases, describe the relationship between volume and temperature of an ideal gas. Explain how this is reflected in your lab data. 4. Pressure and number of moles remained constant during this experiment. If you wanted to test one of these variables in a future experiment, how would you use your knowledge of gas laws to set up the investigation? Which of the following cannot be done through a mobile banking app?Making a depositWithdrawing cashChecking an account balanceTransferring money You have been asked to work on the design of the cover of a new book. The author of the book would like to use a picture of a couple he has taken in the park. What needs to be done to use this image? Someone plezzz help me a difference between bacterial and eukaryotic translation is A research submarine dives at a speed of 100 ft/min directly toward the research lab. How long will it take the submarine to reach the lab from the surface of the ocean?Can you explain why you got the answer too please Suppose you want to estimate the win rate of a slot machine using the Monte Carlo method. MONTE CARLO BET WIN MAX BET After playing the machine for 700 times, the error between the estimated win rate and the actual win rate in the machine's setting is 10-4. If we want to further reduce this error to below 10-?, approximately how many total plays do we need? total number of plays number (rtol=0.01, atol=14-08)